DNPR

Pataasin ang iyong marka sa homework at exams ngayon gamit ang Quizwiz!

A 24-year-old woman is evaluated in the ED for a 5-day history of fever and right knee pain. Over the past 2 days, the left ankle and right wrist have also become painful and swollen. The patient was previously well and has no history of trauma. Her only medication is an oral contraceptive.On physical examination, temp is 101.2oF (38.4oF); other vitals are normal. The right wrist is swollen, erythematous, warm and painful. The dorsum of the right hand is swollen, erythematous and warm, with tenderness to direct palpation and minimal ability to move the fingers. The right knee is tender to palpation with a moderate effusion. The left ankle is tender, and the dorsum of the foot is swollen and tender. Findings of the skin examination is shown below. Which of the following the most like diagnosis? A. Ankylosing spondylitis B. Disseminated gonococcal infection C. Gout D. Rheumatic arthritis E. Staphylococcal arthritis

B, D, E

A 55-year-old woman comes to the physician because of a 1-month history of skin rashes over legs and arms and a 2-month history of irregular pain, numbness and weakness in arms and feet. Current medications include ibuprofen and gabapentin. Her temperature is 37.2°C (99°F), pulse is 90/min, respirations are 20/min and blood pressure is 130/80 mm of Hg. Physical examination shows palpable purpura over the trunk and the extremities, weakness of the ankle dorsiflexors and wrist extensors, and numbness on the dorsum of the hand and the forearm. Laboratory studies show RBCs and RBC casts in urine. Which of the following laboratory tests is the most appropriate next step in evaluating this patient ? A. Coombs test B. ANCA C. Anti-histone antibodies D. Lupus anticoagulant E. Anti-streptolysin O titers

B. ANCA

1. A 35-year-old man comes to the physician because of a 9-month history of itchy, erythematous papules in his extremities and buttocks. His vital signs are within normal limits. Physical examination shows erythematous plaques with white scales in the extensor aspect of his elbows, knees, buttocks, and scalp. Removing the scale leaves a bleeding spot. His fingernails show pitting and onycholysis. A KOH preparation of the nail scrapings shows no hyphae under light microscopy. Which of the following histologic findings is most likely in a skin biopsy of this patient's lesion? A. Acantholysis B. Acanthosis C. Thinning of stratum spinosum D. Hyperplasia of the stratum basale E. Spongiosis

B. Acanthosis

5. A 15-year-old girl is brought to the physician for a routine examination. She is concerned about gaining weight since she plans to be a model. When dining outside her home, she usually restricts her food intake; however, she consumes a large amount in a short period of time, usually when she gets home, but feels guilty soon after. She has a history of cutting her forearms with the metal tab from a soda can. Her last menstrual period was 3 weeks ago. Her BMI is 19 kg/m2. Physical examination shows enlarged salivary glands. Laboratory studies show hypochloremic, hypokalemic metabolic alkalosis. The remainder of the examination shows no abnormalities. Which of the following is the most likely diagnosis? A. Binge eating disorder B. Bulimia nervosa C. Obsessive compulsive disorder D. Body dysmorphic disorder E. Anorexia nervosa

B. Bulimia nervosa

A 24-year-old woman comes to the physician because she is worried about gaining weight. She reports that at least twice a week over the past 5 months, she has eaten excessive amounts of food but feels ashamed and depressed about it after. She is very active in her high school's tennis team and goes running daily to lose weight. Her last menstrual period was 3 weeks ago. BMI is 22.6 kg/m2. Physical examination shows enlarged, firm parotid glands bilaterally. There are erosions of the enamel on the lingual surfaces of the teeth. Which of the following is the most likely diagnosis? A. Obsessive-compulsive disorder B. Bulimia nervosa C. Anorexia nervosa D. Binge eating disorder E. Body dysmorphic disorder

B. Bulimia nervosa

3. A 25-year-old man is brought to the emergency department because of a 1-week history of progressive shortness of breath and a 2-week history of leg weakness which started from the feet and has recently ascended to the thighs. He fell once while walking to work about 10 days ago. He had an episode of bloody diarrhea after a meal of boiled rice and chicken 2 months ago. Neurologic examination shows decreased muscle strength, hyporeflexia, and impaired proprioception, vibration, and fine touch sensations in both legs. An infection with a bacterial agent is thought to have triggered this patient's condition. Which of the following laboratory features are most likely of the causal agent? A. alpha-hemolytic, encapsulated, Gram-positive cocci that produce an IgA protease B. Comma-shaped, oxidase-positive, Gram-negative bacteria that can be grown at 42°C C. Non-lactose fermenting, oxidase-positive, Gram-negative, aerobic bacilli D. Rod-shaped, Gram-positive, spore-forming anaerobe that produces a heat-labile toxin E. Spiral-shaped bacteria with axial filaments, visualized using dark-field microscopy

B. Comma-shaped, oxidase-positive, Gram-negative bacteria that can be grown at 42°C

A 25-year-old woman comes to the physician because of a 5-month history of malaise, intermittent joint pains, weight loss, SOB and occasional fever. Her temperature is 37.0°C (98.6°F), pulse is 100/min, respirations are 20/min, and blood pressure is 140/95 mmHg. Physical examination shows malar rash, oral ulcers, and non-blanching purpuric papules on her legs. .Laboratory tests show anemia, proteinuria and hematuria. The serum ANA test result is positive at a titer of 1:1024. Which of the following mechanisms is most likely involved in the pathogenesis of her disease? A. Antiphospholipid antibodies B. Defective clearance of apoptotic nuclei C. Increased production of IFN-γ D. Molecular mimicry E. Activation of mast cells

B. Defective clearance of apoptotic nuclei

A 35-year-old man comes to the emergency department because of severe left leg pain several hours after injuring himself on a gardening tool. His temperature is 39°C (102.2°F) and his pulse is 105/min. Physical examination of the left leg shows a small laceration on the ankle surrounded by dusky skin and overlying bullae extending to the posterior thigh. There is a crackling sound when the skin is palpated. Surgical exploration shows necrosis of the gastrocnemius muscles and surrounding tissues. Tissue culture shows anaerobic gram-positive rods and a double zone of hemolysis on blood agar. Which of the following best describes the mechanism of cellular damage caused by the responsible pathogen? A. Lipopolysaccharide-induced complement and macrophage activation B. Degradation of cell membranes by phospholipase C. Increase of intracellular cAMP by adenylate cyclase D. Inactivation of 60s ribosome subunit by N-glycosidase E. Inhibition of neurotransmitter release by protease

B. Degradation of cell membranes by phospholipase

A 25-year-old woman comes to the physician because of a 5-month history of malaise, intermittent joint pains, weight loss, SOB and occasional fever. Her temperature is 37.0°C (98.6°F), pulse is 100/min, respirations are 20/min, and blood pressure is 140/95 mmHg. Physical examination shows malar rash, oral ulcers, and non-blanching purpuric papules on her legs. The physician orders anti-nuclear antibody test (ANA) and explains her the concept of "self" and "non-self". In this regard, which of the following most likely describes the importance of "recognition of molecules belonging to self" by the immune system? A. Activate natural killer cells of the innate immune system. B. Determine the safety of interacting with the molecule. C. Induce somatic generation of a B- or T-lymphocyte receptor for the molecule. D. Stimulate binding by pattern recognition receptors. E. Trigger an attack on the cell expressing the self-molecule.

B. Determine the safety of interacting with the molecule.

10. A 90-year-old man comes to his physician to seek an admission in a personal care home. Physical examination is unremarkable. However, physician finds multiple functional impairments that are likely to occur in an elderly person. Which of the following impairments is termed as basic activity of daily living (ADL)? A. Difficulty in using telephone B. Difficulty in taking bath C. Difficulty in cooking D. Difficulty in using public transport E. Difficulty in managing medication

B. Difficulty in taking bath

Clicker Question: A 66-year-old man is brought to the physician by his wife because of a 3-month history of problems walking. He has been going progressively slow when they go for a morning walk and that he has lost interest in their daily activities. His vital signs are WNL. PE shows masked facies and tremors of the right hand when rested on the patient's lap. Neuro exam shows rigidity on passive movement of the right arm. Brain MRI is normal. His symptoms are best explained due to a deficiency of a neurotransmitter. Which of the following enzymes is most directly involved in the formation of this neurotransmitter? A. Phenylethanolamine N-methyl transferase B. Dopa decarboxylase C. Dopamine beta-hydroxylase D. Tyrosine hydroxylase

B. Dopa decarboxylase

A 71-year-old man with a history of rheumatoid arthritis comes to the physician complaining of weakness 1 day after a motor vehicle accident. Physical examination reveals intact sensation and strength in the lower extremities but weakness in the upper extremities bilaterally. The patient is able to move his arms in parallel to the ground but is unable to lift his arms, forearms, or hands upward against gravity. Muscle strength is 2/5 throughout, bilaterally. Ct scan of the cervical spine rules out cervical spine fracture, and MRI demonstrates traumatic C6-C7 disk herniation, buckling of the ligamentum flavum, and edema within the cervical cord in that area. Which of the following tracts is most likely intact in this patient? A. Spinothalamic tract B. Dorsal column medial lemniscus tract C. Corticospinal tract

B. Dorsal column medial lemniscus tract

9. A 42-year-old man comes to the emergency department because of a 2-week history of multiple scores and abscesses on his lower limbs. He stated that these lesions do not appear to be healing. He is not aware of how these lesions developed and is concerned that it might be something serious. He was treated on two occasions in the past year for similar skin infections. On one occasion, the patient was hospitalized for treatment of his infection. A swab and culture of the lesion show the presence of fecal bacteria. Which of the following is the most likely cause of this patient's condition? A. Conversion disorder B. Factitious disorder C. Malingering D. Illness anxiety disorder E. Somatic symptom disorder

B. Factitious disorder

Janet a 24 yo with Type one Diabetes presents to the ED with ketotic breath, mild tachypnea, and increased urine output. Although sodium concentration is the major determinant of plasma osmolarity, other compounds can contribute to total solute load. Excessive plasma concentrations of which of the following compounds is most likely to lead to the development of an osmotic gradient in this patient? A. water B. Glucose. C. Lactate. D. Potassium. E. Urea.

B. Glucose.

A 24-year-old woman comes to the physician because of a 2-day history of fever and painful blisters in her genitalia. Physical examination shows group of tender vesicles on an erythematous base in the labia on the left side of her vulva. Different types of Human Herpes virus (HHV) types are matched with the disease caused in human. Which of the following correctly matches with this patient? HHV Type Disease caused in Human A. HHV-1 Herpes simplex infection B. HHV-2 Genital herpes C. HHV-3 Chicken pox & shingles D. HHV-4 Infectious Mononucleosis E. HHV-5 Pneumonia in AIDS patient F. HHV-6 & 7 Roseola infantum G. HHV-8 Kaposi sarcoma

B. HHV-2 Genital herpes

A 32-year-old woman comes to the physician because of increasing back pain for the past 10 months. The pain is worse in the morning when she wakes up and improves with activity. She used to practice yoga but stopped 5 months ago as bending forward became increasingly difficult. She has also had bilateral hip pain for the past 4 months. She has not had any change in urination. She has celiac disease and eats a gluten-free diet. Her temperature is 37.1°C (98.8°F), pulse is 65/min, respirations are 13/min, and blood pressure is 116/72 mmHg. Physical examination shows the range of spinal flexion is limited. Flexion, abduction, and external rotation of bilateral hips produces pain. Further evaluation of this patient is likely to show which of the following? A. High levels of rheumatoid factor B. HLA-B27 positive genotype C. Presence of anti-dsDNA antibodies D. High levels of creatine phosphokinase E. Presence of anti-Ro and anti-La antibodies F. HLA-DR3 positive genotype

B. HLA-B27 positive genotype

A 54y old hypertensive male presents with sudden onset of headache for 3 days. The headache has been increasing from 2/10 to an 8/10 intensity this morning. While showering this morning he developed one sided weakness of his arm and leg, felt nauseas and vomited. On examination his pupils are unequal, and he seems confused. What is the most likely diagnosis? A. Ischemic stroke B. Hemorrhagic stroke C. Brain tumor D. Brain abscess

B. Hemorrhagic stroke

7. A 5-year-old boy is brought to the pediatrician because of a 2-week history of morning headaches, vomiting, and gait problems. At the visit, the mother reports that her son's symptoms has worsened in less than a week. Physical examination reveals truncal ataxia and gait imbalance. The child falls to the ground when asked to walk following a straight line. Brain MRI is shown. The patient is treated with complete surgical excision followed by radiotherapy and chemotherapy. Biopsy results showed a Medulloblastoma. Which of the following is most likely present on the biopsy? A. Rosenthal fibers B. Homer-Wright rosettes C. Perivascular pseudorosettes D. Psammoma bodies E. Pseudopalisades

B. Homer-Wright rosettes

An 81-year-old man is brought with a 2-year history of progressive cognitive impairment in the form of memory loss, personality changes and errors in judgement. His past medical history is unremarkable. MMSE test score is 14. Which of the following changes in his brain is most likely? A. Depletion of dopamine neurotransmitter B. Hyperphosphorylation of neuronal tau protein C. Deposition of Amyloid-A in the brain D. Degeneration of caudate nucleus E. Aggregates of intraneuronal alpha synuclein

B. Hyperphosphorylation of neuronal tau protein

After looking up the last maintenance fluids in the EMR Briana, your 8mo, received for a previous admission to the hospital you started to treat her Dehydration with her fluids of 0.2% Sodium Chloride + D10W. After 1 hour she begins to have a tonic clonic seizure. Following protocols, you administer 0.1mg/kg lorazepam, followed by another dose 5 minutes later. Her seizures still persist. She does not have a history of epilepsy and no fever. What electrolyte abnormality can you assume she has, and if corrected will stop the seizure? A. Hypercalcemia B. Hyponatremia C. Hyperkalemia D. Hypophosphatemia E. hypernatremia

B. Hyponatremia

11. An 81-year-old woman comes to the physician because of a 6-month history of involuntary leakage of urine. She wets her underwear during maneuvers such as coughing, laughing or changing her position while sitting. There is no history of urinary infections, diabetes or hypertension. Her vital signs are within normal limits. Physical examination shows no abnormalities. Postvoid residual urine measurement is within normal limits. Which of the following pathophysiologic mechanisms is most likely the basis of this patient's incontinence? A. Obstruction to the urethra by fibrosis B. Incompetence of the external urethral sphincter C. Inability of the brain to sense bladder filling D. Overactivity of the detrusor muscles E. Underactivity of the detrusor muscles

B. Incompetence of the external urethral sphincter

A 34-year-old woman comes to the physician because of a 2-month history of fatigue, amenorrhea, and facial plethora. She has a 3-year history of systemic lupus erythematosus. Current medications are azathioprine and high dose corticosteroids. Her blood pressure is 150/90 mm Hg. Laboratory studies show : Urinary cortisol 235 μg/24 h (normal 3.5-45 μg/24 h), ACTH 1.2 pg/ml (normal range 7.2-64 pg/ml). Which of the following alterations in the total body fluids and serum sodium concentration is most likely in this patient? A. Increased body fluid and decreased serum sodium concentration. B. Increased body fluid and normal/high serum sodium concentration. C. Decreased body fluid and normal/high serum sodium concentration. D. Decreased body fluid and increased serum sodium concentration. E. Decreased body fluid and decreased serum sodium concentration.

B. Increased body fluid and normal/high serum sodium concentration. -Cushing's syndrome with water retention

An outbreak of corona virus disease occurs in the USA. The high infectivity of the virus causes a rapid increase in patients presenting with symptoms that range from fatigue, Fever or chills, cough, difficulty breathing to gastrointestinal symptoms, which results in shortage of medical supplies. The number of patients that arrive at the local hospitals exceeds the amount of medical supplies needed for standard treatment, intensive care, and precautionary measures. Which of the following ethical principles is most appropriate in addressing this situation? A. Autonomy B. Justice C. Beneficence D. Nonmaleficence

B. Justice

9. A 39-year-old woman comes to the physician because of a 6-week history of progressive increase in the size of a mole on her left leg. She had a mole in this region since birth but the mole has recently increased in size. Physical examination shows a 6-mm diameter bluish black pedunculated lesion on her left leg. The lesion shows a bleeding spot. Which of the following features best characterizes this lesion? A. Mostly associated with repeated sunburns B. Lesions sometimes are amelanotic C. Most common in females than males D. Lesion arises from a lentigo maligna E. Shows positive Hutchinson sign

B. Lesions sometimes are amelanotic

7. A 35-year-old woman comes to her family doctor because of fatigue, dizziness, headaches, irregular menses, and environmental allergies for the past month. She has a past history of anorexia nervosa. Each day she use about 25 laxatives, exercises for several hours, and eats salad and toast. Her temperature is 37C (98.6F), pulse 90/min, blood pressure 90/60 mm Hg; at 5'5", weighs 92 pounds. She demonstrates a good understanding of the diagnosis and the recommended therapy for anorexia but refuses any medical intervention. Which of the following action by the physician is most appropriate in this situation? A. Admit the patient for treatment since she is incompetent B. Maintain a therapeutic relationship with ongoing dialogue C. Refer the patient to a psychologist for counseling D. Do not apply the principle of autonomy in this situation E. Treat the patient for her current symptoms and send her home

B. Maintain a therapeutic relationship with ongoing dialogue

7. A 10-month-old boy is brought to the physician by his mother because of a 2-week history of pruritic red lesions on his cheeks and runny nose. There is no history of fever. His 4-year-old brother has similar lesions and asthma since birth. Physical examination shows red, scaly, excoriated cheeks as well as generalized dryness with scattered excoriations in antecubital fossae, popliteal fossae. Pathogenesis of this patient's condition is most likely related to which of the following? A. Autoantibodies to hemidesmosomes B. Mutation in filaggrin gene C. Autoantibodies to desmosomes D. Immune complex deposition in the dermal vessels E. Increased cathelicidin formation in the skin

B. Mutation in filaggrin gene

A 14-year-old female comes to the physician because of a 6-week history of 8 pounds(3.6kg) unintentional weight loss. She also has a 2- months history of excessive thirst, excessive urination, and need to arise during the night for urination. Laboratory tests show a fasting blood glucose of 460 mg/dL(normal: 70 to 100 mg/dL). The patient is diagnosed with an autoimmune disease. The physician tries to explains her the concept of autoimmunity, informing that her T-cells likely recognize self-antigens due to defective development in the thymus. Which of the following processes is the physician most likely referring to? A. Positive selection B. Negative selection C. Suppression D. Peripheral tolerance E. Epitope spreading

B. Negative selection

A 4-year-old girl is brought to the physician by her mother because of a 3-month history of excessive urination and muscle weakness. The mother says she is drinking large amounts of water and is always thirsty. Her weight is in 10th percentile and her height is the fifth percentile. Laboratory studies show: Serum Sodium : 135 mEq/L (136-146 mEq/L ) Serum Potassium : 2.2 mEq/L (3.5-5 mEq/L ) Serum Chloride : 107 mEq/L (95-105 mEq/L) Serum Bicarbonate : 19 mEq/L (22-28 mEq/L) Serum Glucose(fasting): 82 mg/dL (70-100 mg/dL) Serum Phosphate: 1.9 mg/dL (3.0-4.5 mg/dL)Urinalysis shows phosphaturia, glucosuria, and aminoaciduria. Arterial blood gas analysis on room air shows a pH of 7.31, PCO2 of 32 mm Hg, and PO2 of 105 mm Hg. Which of the following primary acid-base disorders is most likely in this patient? A. Metabolic alkalosis B. Non anion gap metabolic acidosis C. Anion gap metabolic acidosis D. Respiratory alkalosis E. Respiratory acidosis

B. Non anion gap metabolic acidosis (fanconi syndrome)

Fluids that are ISO tonic can be given as a fluid bolus to those with dehydration, in shock, hypoglycemic, or many other reasons you may need to fluid bolus a patient. Hypotonic fluids should never be bolused, while hypertonic ones can be. Which fluid below is NOT isotonic A. D5W (5%Dextrose in Water) B. O.45% Normal Saline C. 0.9% NaCl D. Lactated Ringers E. Plasma-Lyte 148 F. 5% Albumin

B. O.45% Normal Saline

A 75 year old female with a history of advanced Alzheimer's disease is brought to the emergency department because of confusion. The patient lives at a nursing home, and the nurse accompanying her states that the patient has become increasingly confused and irritable over the past two days. She states that the patient developed a cough, productive sputum four days ago and has had decreased oral intake since that time. Past medical history is significant for Alzheimer's disease, hypertension, arthritis, and a hip fracture. Current medications include rivastigmine, hydrochlorothiazide, acetaminophen, and a multivitamin. Physical examination revealsT=37.8' (100.0') , Bp 98/60, Pulse 114/minthe oropharynx reveals dry, mucous membranes, and pharyngeal injection. Poor skin turgor is noted. Which of the following is most appropriate next step in the patient care? A. Measure the FeNa B. Obtain a Urine Osmolality C. MRI of the brain D. Oral rehydration therapy with water E. Water deprivation test

B. Obtain a Urine Osmolality

11. A 25-year-old woman comes to her primary care physician because of intermittent episodes of heart palpitations accompanied by diaphoresis, trembling, and lightheadedness while reading at home. The symptoms first occurred during a class presentation. The second happened while on a family picnic. She fears that "the feelings will keep coming out of nowhere," and she constantly worries about having another episode. Her last episode was two months ago. She denies the use of prescription medications, illicit drugs, and alcohol. Physical examination and laboratory studies are within normal limits. Which of the following is the most likely diagnosis? A. Performance anxiety B. Panic disorder C. Social phobia D. Agoraphobia E. Generalized anxiety disorder

B. Panic disorder

A 60-year-old woman comes to the physician because of a 1-month history of left-sided headache and blurred vision. The pain is worse when performing tasks like combing hair or working in the kitchen. Her temperature is 37.2°C (99°F), pulse is 80/min, respirations are 16/min and blood pressure is 130/80 mm of Hg. Physical examination shows a tender nodular thickening on the left side of the forehead with scalp tenderness and neck rigidity. Laboratory studies show raised erythrocyte sedimentation rate and C-reactive protein. Which of the following is the most likely additional finding in this patient? A. Allergic rhinitis B. Polymyalgia rheumatica C. Rapidly progressive glomerulonephritis D. Cavitations in lung E. Arterial bruits

B. Polymyalgia rheumatica

8. A 34-year-old man comes to the physician because of a 2-month history of difficulty concentrating at work. He feels hopeless and is worried he may lose his job due to poor performance. He feels constantly tired but attributes his fatigue to waking up very early and being unable to fall back asleep. He speaks more slowly than usual, and his appetite has decreased. He used to enjoy swimming but has not been as interested in this recently. He lives alone and was recently discharged from hospitalized after being admitted for 3 months for depression. He states that he has thought about ending his life, but he has no specific plan. He drinks 2-3 cans of beer daily. Which of the following is the highest risk for suicide in this patient? A. Being single or living alone​ B. Recent discharge from hospital C. Prominent hopelessness D. Male sex​ E. ​Daily use of alcohol

B. Recent discharge from hospital

6. A 79-year-old man is admitted to the hospital with a massive intracranial bleed. He has been placed on a ventilator due to respiratory failure associated with brain stem herniation. When you try to remove him from the ventilator, there are no spontaneous respiratory efforts by the patient. His blood pressure and pulse are not recordable. The patient makes no purposeful movements. There is no pupillary reaction to light; no nysatagums on cold caloric testing; oculocephalic and corneal reflexes are absent; EEG shows electrocerebral silence. This condition continues for more than 10 hours. Which of the following is the most appropriate next action for this patient? A. Place a nasogastric tube to prevent aspiration B. Remove the ventilator support C. Get a court order authorizing you to remove the ventilator D. Do cerebral angiography to aid diagnosis E. Continue the ventilator support

B. Remove the ventilator support

A 9-month-old boy is brought to the physician because of a 3-day history of low-grade fever, runny nose and sore throat and, 1-day history of a skin rash. Physical examination shows an erythematous macules and papules on his face, trunk and extremities and tender cervical lymphadenopathy. A. Measles B. Rubella C. Adenovirus infection D. Enterovirus E. EBV F. Toxin-mediated eruptions

B. Rubella

A 6-year-old girl is brought to the physician because of intermittent fevers and painful swelling of the left ankle for 2 weeks. She has no history of trauma to the ankle. She has a history of sickle cell disease. Current medications include hydroxyurea and acetaminophen for pain. Her temperature is 38.4°C (101.2°F) and pulse is 112/min. Physical examination shows a tender, swollen, and erythematous left ankle with point tenderness over the medial malleolus. A bone biopsy culture confirms the diagnosis. Which of the following is the most likely causal organism? A. Pseudomonas aeruginosa B. Salmonella typhi C. Escherichia coli D. Streptococcus pyogenes E. Streptococcus pneumoniae

B. Salmonella typhi

A 42-year-old woman comes to the physician because of a 2-month history of progressive muscular weakness. She has had difficulty climbing stairs, getting up from chairs, and brushing her hair. Her vital signs are within normal limits. Muscle strength is 2/5 with flexion of the hips and 3/5 with abduction of the shoulders. She is unable to stand up from her chair without the use of her arms for support. Laboratory studies show elevations in leukocyte count, erythrocyte sedimentation rate, and creatine kinase concentration. Histological evaluation of a biopsy specimen of the deltoid muscle is most likely to show which of the following? A. Muscle fiber necrosis with rare inflammatory cells B. Sarcolemmal MHC-I overexpression with CD8+ lymphocytic infiltration C. Sarcoplasmic rimmed vacuoles with CD8+ lymphocytic infiltration D. Relative atrophy of type II muscle fibers with hypertrophy of type I muscle fiber E. Perimysial inflammation with perivascular CD4+ T lymphocytic infiltration

B. Sarcolemmal MHC-I overexpression with CD8+ lymphocytic infiltration

6. A 7-year-old boy is brought to the physician by his mother because of a 1-year history of declining academic performance and poor language skills. He had a normal weight and length at birth but is noted to have a declining growth rate. He is at the 25th percentile for height. Physical examination shows iris hamartomas on both eyes and multiple freckles in the axillary and inguinal regions bilaterally. The physician's younger brother has similar skin lesions that appear as hyperpigmented intracutaneous plaques with excess hair, and he recently underwent cataract surgery. The physician considers the same clinical condition for the patient and his brother but with varying phenotypical expressions of the disease. Which of the following findings is most likely expected in the physician's brother? A. Chromosome 17 aberration B. Slow-growing tumors on cranial nerve VIII C. Adenoma sebaceum D. Facial nevus flammeus E. Dilated blood vessels on the bulbar conjunctivae

B. Slow-growing tumors on cranial nerve VIII

A 5-year-old boy who is previously healthy is brought to the emergency department because of fever, irritability, malaise, and left knee pain for 4 days. Four days ago, he fell off his bike and scraped his knee. His temperature is 39.1°C (102.4°F). The patient walks with a limp. Physical examination shows swelling and point tenderness over the medial aspect of the left knee. An MRI of the left knee shows edema of the bone marrow and destruction of the medial metaphysis of the tibia. Which of the following is the most likely causal organism? A. Staphylococcus epidermidis B. Staphylococcus aureus C. Pseudomonas aeruginosa D. Pasteurella multocida E. Streptococcus pyogenes

B. Staphylococcus aureus

A 50-year-old woman comes to the physician because of a 6-month history of progressive difficulty in swallowing food, heart burn and a 5-kg weight loss. She has a 1-year history of her fingers turning blue when exposed to cold. Physical examination shows a mask-like, taut skin over the face , arms, abdomen and legs with multiple spider-like discolorations. Laboratory tests show positive ANA 1:512 titer with a nucleolar pattern and positive anti-DNA topoisomerase 1 antibody 1:1024 titer. Serum complement levels are normal. The actions of which of the following cytokines is most likely related to the pathogenesis of this disease? A. Type 1 interferon B. TGF-β C. IL-17 D. IL-2 E. IFN-gamma

B. TGF-β

Janet your patient who presented in DKA, who you successfully treated through fluid replacement over 48 hours, and insulin, now happy , feeling better, ready to eat, with normal vital signs and has no anion gap (from the ketones , also no longer in the urine). Unfortunately, the endocrinology fellow insisted on a blood gas before starting their regular regimen of insulin and diet, to make sure there was no acidosis. The ABG, which made the patient very angry showed: 7.28/ 40/ 120/ 16 / -9.6 What can you tell the fellow is the most likely cause for this result? A. We need to keep on insulin until the ketones resolve. B. This is expected hyperchloremic acidosis and it will clear with normal renal function. C. She likely has a lactic acidosis from septic shock. D. We should probably repeat the labs for accuracy. E. She has a previously undiagnosed renal tubular acidosis.

B. This is expected hyperchloremic acidosis and it will clear with normal renal function.

8. A 35-year-old man is brought to the emergency department after being involved in a high-speed motor vehicle collision. On arrival, he is unconscious. His pulse is 138/min, respirations are 13/min and shallow, and blood pressure is 69/56 mm Hg. Ultrasound shows blood in the left upper quadrant of the abdomen, and the x-ray shows a fracture of the right femur. His hemoglobin concentration is 7.2 g/dL and his hematocrit is 21%. The surgeon decided to move the patient to the operating room for an emergent explorative laparotomy. Packed red blood cell transfusion is ordered prior to surgery. The patient is a Jehovah's Witness. The patient has no advance directives. The patient's next of kin cannot be contacted. Which of the following is the most appropriate next best step in management? A. Administer only 0.9% saline solution B. Transfusion of packed red blood cells C. Consult hospital ethics committee D. Administer high-dose iron dextran E. Proceed to surgery without transfusion

B. Transfusion of packed red blood cells

10. A 55-year-old man presents to the emergency room because of anxiety, difficulty falling asleep, nausea, and vomiting. He is a heavy drinker, having consumed half a bottle of Whiskey almost every day for a decade. He stated that he started feeling unwell after he stopped using alcohol less than two days ago but admitted he has a strong desire to return to the bottle. His blood pressure is 140/90 mm Hg, pulse is 115/min, respirations are 18/min, and temperature is 37.6°C (99.7°F). On physical examination, the patient is diaphoretic and agitated. What is additional clinical manifestation is expected in this patient? A. Seizures B. Tremors C. Sensation of bugs crawling on his skin D. Fever E. Confusion

B. Tremors

A 33-year-old man comes to the emergency department because of a 1-day history of fever, nausea, and severe scrotal pain. Medical history is remarkable for type 1 diabetes mellitus. He is sexually active with multiple male and female partners and uses condoms inconsistently. His only medication is insulin. His temperature is 39.3ºC (102.7ºF), pulse is 115/min, respirations are 17/min, and blood pressure is 87/50 mm Hg. Physical examination shows a tender, diffusely swollen scrotum with dusky skin coloration that extends to the penis and perineum. There is crepitus on light palpation. Abdominal examination shows no abnormalities. Laboratory studies show a leukocyte count of 18,400/mm3 and a C-reactive protein concentration of 46.4 mg/L (N = 0.08-3.1). Which of the following is the most appropriate next step in management? A. Open hernia repair with bowel resection B. Gram stain and culture from the affected tissue C. Aggressive debridement of the affected tissue D. Oral broad spectrum antibiotic therapy E. Duplex ultrasound of the scrotum

C. Aggressive debridement of the affected tissue

2. A 20-year-old woman is brought to the emergency department because of a 40-minute history of generalized body itching associated with skin rash. She says that these symptoms started about 25 minutes after a meal containing strawberries and shellfish. She has a family history of C1 esterase inhibitor deficiency. Her pulse is 112/min and blood pressure is 100/70 mmHg; other vital signs are within normal limits. Physical examination shows scattered palpable, blanching plaques surrounded by erythematous skin on the trunk and extremities. Scratching her skin with a blunt object produces similar lesions on a normal skin. This patient is at the greatest risk of developing which of the following? A. Toxic epidermal necrolysis B. Rapidly progressive glomerulonephritis C. Angio-edema of the larynx D. Disseminated intravascular coagulation E. Cerebral edema

C. Angio-edema of the larynx

A 57-year-old man comes to the physician because of a 10-day history of facial rash and joint pains. He has a 5-year history of hypertension and angina. Current medications are amiloride, hydralazine, metoprolol and enalapril. His last medication change was 4 weeks ago. Physical examination shows tenderness and erythema over his knees, proximal interphalangeal and metacarpophalangeal joints. Laboratory tests show positive ANA and CRP. Complete blood counts, liver function tests, and renal function tests are normal. Which of the following antibodies is most likely present in this patient? A. Anti-dsDNA B. Anti-CCP C. Anti-histone D. ANCA E. Anti-phospholipid

C. Anti-histone

A 30-year-old woman comes to the physician because of a 3-month history of fatigue, tremors, palpitations and heat intolerance. Her temperature is 37°C (98.6°F), pulse is 110/min, respirations are 16/min and blood pressure is 140/90 mm of Hg. Physical examination shows warm and moist skin, proptosis and diffusely enlarged thyroid gland. Laboratory studies show normocytic anemia, low serum cholesterol and positive autoantibodies. Which of the following mechanisms best explains the pathogenesis of this patient's condition? A. Mutations in AIRE gene B. Immune complex deposition in the thyroid gland C. Antibody-mediated stimulation of the TSH-receptors D. Antibody-mediated inhibition of the TSH-receptors E. Antibody-mediated stimulation of TSH secretion

C. Antibody-mediated stimulation of the TSH-receptors

5. A 7-year-old boy is brought to the emergency department by his mother because of a 2-day history of high-grade fever and blisters in his right arm. There is no history of trauma to the arm. He recovered from chickenpox 10 days ago. His 12-year-old sister has recently had a sore throat and fever. Physical examination shows multiple, thin-roofed, vesicles filled with a cloudy, dark yellow fluid. A swab from the base of an unroofed blister is cultured in the laboratory and shows growth of a Gram-positive colonies in chains. Which of the following additional cultural characteristics are most likely to be exhibited by the causal agent? A. Alpha hemolysis and Bacitracin resistant B. Alpha hemolysis and Bacitracin sensitive C. Beta hemolysis and pyrrolidonyl arylamidase (PYR) positive D. Beta hemolysis and Catalase positive E. Beta hemolysis and Optochin resistant

C. Beta hemolysis and pyrrolidonyl arylamidase (PYR) positive

A 62-year-old man comes to the physician because of a 1-day history of nausea, confusion, and lethargy. He has a 5-year history of hypertension, and a 6-month history of lung cancer for which he is undergoing treatment. Physical examination shows moist mucous membranes and normal skin turgor. There is no pedal edema. He is oriented to person but not to time and place. Laboratory studies show :Serum Sodium : 112 mEq/dL (136-146 mEq/L ) Serum Potassium: 4.0 mEq/dL (3.5-5 mEq/L )Serum Osmolarity: 250 mOsm/kg (275-295 mOsm/kg) Urine Osmolarity: 550 mOsm/kg (50-1200 mOsmol/kg)Which of the following is the most appropriate next step in management? A. Slow intravenous infusion of lactated ringer's solution B. Slow intravenous infusion of 0.9% normal saline C. Bolus intravenous infusion of 3% normal saline D. Slow intravenous infusion of 5% dextrose in water (D5W solution) E. Slow intravenous infusion of 23.4% normal saline

C. Bolus intravenous infusion of 3% normal saline -SIADH

A 22 yo female is in the ICU following elective scoliosis surgery for a cobb angle of 42. the cell saver unit broke at the beginning and therefore received six units of packed red blood instead of her own blood. On post op day 2, her bed side monitor is alarming for a prolonged QT, she is dismayed with some intermittent uncontrolled contraction in hands and feet (carpopedal spasms) and she noted that when she started eating she had numbness around her mouth. A correction of which of the following should be done urgently? A. Magnesium B. Potassium C. Calcium D. Citrate

C. Calcium

A 25-year-old man is brought to the emergency department because of central chest pain for the past 40 minutes. He is diaphoretic, agitated, and confused. His temperature is 39 o c (102.2 O F), his pulse is 130/min, and his blood pressure is 180/90 mm Hg. Examination of the pupils shows mydriasis with minimal reactivity to light. The tone is normal. There is no clonus, but reflexes are brisk. Abdominal examination shows increased bowel sounds. An ECG shows ST elevations in leads VI -V4. The use of which of the following substance is most likely the cause of this patient presentation? A. Phencyclidine B. Cannabis C. Cocaine D. Lysergic acid diethylamide E. Amphetamines

C. Cocaine

1. An 87-year-old woman was admitted to the intensive care unit after she was found lying on the floor at her home. An MRI of the brain shows an extensive thrombus in the left middle cerebral artery. More than a week after admission, she is unable to communicate or safely swallow food. The patient's sister requests the placement of a percutaneous endoscopic gastrostomy (PEG) tube for nutrition. The patient's eldest son declines the intervention. There is no living will, and her husband died two years ago. Which of the following is the most appropriate course of action for the physician? A. Proceed with PEG placement B. Consult the hospital ethics committee C. Encourage a family meeting D. Request a court-appointed guardian E. Transfer to a physician specialized in hospice care

C. Encourage a family meeting

A 32-year-old man comes to the physician because of a 1-day history of an itchy blistering eruption. He was hiking in a national park, 3 days ago. There is no history of exposure to any chemical agent. He says he was wearing T-shirt and full pants during hiking. Physical examination shows linear papules and vesicles on his forearms, dorsum of hands and feet. Which of the following features best characterizes this patient's condition? A. He has an irritant contact dermatitis B. He likely has an allergic contact dermatitis to ragweed C. He likely has a type IV reaction to poison ivy D. He has a type II hypersensitivity E. The eruption is expected to clear in 2-4 days.

C. He likely has a type IV reaction to poison ivy

A 25-year-old woman comes to the physician because of a 5-month history of malaise, intermittent joint pains, weight loss, SOB and occasional fever. Her temperature is 37.0°C (98.6°F), pulse is 100/min, respirations are 20/min, and blood pressure is 140/95 mmHg. Physical examination shows malar rash, oral ulcers, and non-blanching purpuric papules on her legs.Laboratory tests show anemia, proteinuria and hematuria. The serum ANA test result is positive at a titer of 1:1024. She gets pregnant. A complete lupus antibody profile is done. Results are shown: Antinuclear antibodies Positive (titer: 1:1024), speckled pattern Anti-Ro/SSA antibodies - Positive Anti-double-stranded DNA anti-bodies - Negative Anti-U1-ribonucleoprotein antibodies- Negative Anti-Smith anti-bodies- NegativeThe neonate is most likely to be at risk for which of the following complications? A. Pleural effusion B. Libman sach's endocarditis C. Heart block D. Glomerulonephritis E. Non erosive arthritis

C. Heart block

A 52-year-old woman comes to the physician because of pain and stiffness in both of her hands for the past 3 weeks. She complains of early morning joint stiffness lasting more than 1 hour that eases with physical activity. Physical examination shows swelling and tenderness of the metacarpophalangeal joints bilaterally. Range of motion is decreased due to pain. There are subcutaneous, nontender, firm, mobile nodules on the extensor surface of the forearm. What other additional physical examination findings may be present in this patient? A. Clubbing of the fingers and toes B. Skin nodules composed of uric acid C. Hyperextension of the PIP joint with flexion of the DIP joint D. Firm nodules of the distal interphalangeal joints E. Tenderness over the sacroiliac joints

C. Hyperextension of the PIP joint with flexion of the DIP joint

Patting yourself on the back for saving yet again another life, you see Briana's labs confirming your suspicions of hyponatremia. And now you are asked by your attending physician to explain the cause of the electrolyte abnormalities. Which statement best describes your patient. A. No abnormalities B. Severe diarrhea causing an anion gap acidosis C. Large gastric fluid losses causing a hypochloremic metabolic alkalosis D. Post renal Acute kidney injury resulting in Sodium wasting E. AKI and diarrhea causing compensated metabolic acidosis

C. Large gastric fluid losses causing a hypochloremic metabolic alkalosis

2. A 72-year-old man is brought to the emergency department because of a 2-hour history of visual field impairment. He has a 20-year history of hypertension and type 2 diabetes mellitus but is not compliant with medications. His temperature is 37°C (98.6°F), pulse is 110/min and irregularly irregular, and blood pressure is 160/100 mmHg. Physical examination of the four extremities is normal. A head CT scan is performed 72 hours after the onset. Which of the following findings is most likely in this patient? A. Bitemporal hemianopia B. Right homonymous hemianopia with macular sparing C. Left homonymous hemianopia with macular sparing D. Right lower extremity weakness E. Left lower extremity weakness F. Right face and upper extremity weakness

C. Left homonymous hemianopia with macular sparing

A 10-year-old boy started to be more clumsy than usual and had trouble keeping his balance when he was running on rough ground or up and down stairs. These symptoms became worse over a 10-month period, and he occasionally fell to the right. Exam showed dysmetria in the right upper and lower extremities. For the last 2 weeks he has been experiencing headaches. This morning he developed a grand mal seizure. What is the next step in your treatment plan after doing your physical examination? A. CBC B. Physiotherapy C. MRI brain D. Start on Phenytoin E. CSF analysis

C. MRI brain

8mo female with an extensive medical hx presents with no urine output for one day. Mother states that since discharge she has been having 3-6 ounces of bilious output through her G- tube (gastrostomy tube). Today it was 8 ounces. She has been administering feeds via the Jujenostomy tube (j-tube). She has continued to have stools. Today her stool was gray colored. She has not been irritable; she has not had fever. No URI signs or symptom. She has had No difficulty breathing. She appeared more depressed today to her mother which prompted her to bring Briana to the ED. In the ED she was noted to have sunken eyes, doughy skin and delayed capillary refill. VS: HR 145, BP 100/60, RR 35, sat 100% Afebrile. She weighed 5.6kg at her last drs office visit and today is 5 kg. What degree of dehydration is presenting? A. None - she is not dehydrated B. Severe ~15% C. Moderate ~10% D. Mild ~5%

C. Moderate ~10%

1. A tall, thin 45-year-old man is brought to the emergency department because of a 30-minutes history of severe headache. The patient tells the physician that the headache is the worst of his life. The headache started suddenly while eating breakfast this morning. Physical examination shows nuchal rigidity and right anisocoria. Head CT scan is shown in the image. The patient was managed in the Intensive Care Unit and stayed conscious. After four days of the onset the patient started with left sided hemiparesis. Which of the following drugs is most likely to be used for this specific complication? A. Aspirin B. Heparin C. Nimodipine D. Sumatriptan E. Amitriptyline

C. Nimodipine

A 39-year-old woman is brought to the physician by her husband because she does not want to get out of bed and has been intermittently crying all day. Her symptoms started after she was fired two days ago from her job of 10 years. She says she feels sad and does not understand what she will do with her life now. Her menses started one week ago. She appears sad and very upset. Her temperature is 37°C (98.6°F), pulse is 102/min, respirations are 20/min, and blood pressure is 130/80 mm Hg. Physical examination shows no other abnormalities. The mental status examination is normal. She maintains eye contact and follows instructions. Which is the most likely explanation for this patient's findings? A. Major depressive disorder B. Acute stress disorder C. Normal stress reaction D. Adjustment disorder E. Persistent depressive disorder F. Bipolar disorder G. Complicated grief H. Premenstrual dysphoric disorder

C. Normal stress reaction

What is the likely cause for this patient'shypercalcemia? A. Too many supplements B. Cell Lysis from chemotherapy C. Parathyroid hormone-related peptide D. A new tumor in the parathyroid gland from radiation therapy E. Excessive red blood cell transfusions for anemia

C. Parathyroid hormone-related peptide

11. A 3-year-old boy is brought to the pediatrician because of a 4-day history of headache, vomiting and gait impairment. Physical examination shows ataxia. Fundoscopic examination shows papilledema. Brain MRI shows a 3-cm, well-circumscribed mass in the fourth ventricle with enlargement of the lateral and third ventricles (image shown). The mass is surgically removed, and histologic examination of the mass shows an ependymoma. Which of the following characteristics is most likely present in the biopsy? A. GFAP + B. Fried egg appearance C. Perivascular pseudorosettes D. Rosenthal fibers E. Pseudopalisades F. Psammoma bodies

C. Perivascular pseudorosettes

A 21-year-old man is brought to the emergency department by his friend because of strange behavior. Three hours ago, he was at a college party where he got involved in a fight with the bartender. His friend says he smoked a cigarette before becoming irritable and combative. He repeatedly asked, "Why are you pouring urine in my drink?" before hitting the bartender. He has no history of psychiatric illness. His temperature is 38°C (100.4°F), his pulse is 100/min, his respirations are 19/min, and his blood pressure is 158/95 mm Hg. Examination shows muscle rigidity. He has a reduced degree of facial expression. He has no recollection of his confrontation with the bartender. Which of the following substance is most likely responsible for this patient's presentation? A. Heroin B. Cocaine C. Phencyclidine D. Alcohol E. Marijuana

C. Phencyclidine

Which of the following is the most common clinical pattern of Rheumatoid arthritis? A. Monoarticular, chronic, degenerative arthropathy B. Oligoarticular, symmetrical, acute, inflammatory arthropathy C. Polyarticular, symmetrical, chronic, inflammatory arthropathy D. Polyarticular, asymmetrical, chronic, degenerative arthropathy

C. Polyarticular, symmetrical, chronic, inflammatory arthropathy

4. A 22-year-old woman comes to the physician accompanied by her mother because of difficulty falling asleep. Her mother states she is concerned that her daughter is not the same since she witnessed her boyfriend being shot at a gas station during a robbery three months ago. Her mood now fluctuates between really tense and on edge to detached and numb. She states that she cannot fall asleep at night because, as soon as she closes her eyes, she sees the scene of her boyfriend in a pool of blood. She also has nightmares of the event. During the day, she is hypersensitive to loud sounds and reexperiences the event whenever she sees a gas station. She is a social drinker and had used marijuana a few days before the incident. Mental status examination shows a depressed mood and a blunted affect. Which of the following is the most likely diagnosis? A. Acute stress disorder B. Adjustment disorder C. Post traumatic stress disorder D. Major depressive disorder E. Panic disorder

C. Post traumatic stress disorder

2. A 52-year-old man comes to the physician because of a 3-month history of lower leg weakness that worsens with exertion and a 2-month history of a 5-kg (11-lb) weight loss, fatigue, and difficulty swallowing. His vital signs are within normal limits. Physical examination shows facial swelling and right eye ptosis. Electrodiagnostic studies of the leg muscle show a decremental response to repetitive motor nerve stimulation. Which of the following mechanisms is the most likely cause of this patient's presentation? A. Duplication of the peripheral myelin protein-22 gene B. Motor neuron damage from abnormal SOD1 accumulation C. Production of antibodies against the acetylcholine receptor at the neuromuscular synapse D. Production of antibodies against the voltage-dependent calcium channels at the neuromuscular synapse E. Muscle loss from reduced or absent dystrophin

C. Production of antibodies against the acetylcholine receptor at the neuromuscular synapse

A 42-year-old man is evaluated for a 1-month history of a painful, swollen right finger and a swollen left toe. He has no other symptoms and generally feels well. He has not noticed a skin rash. Medical history is unremarkable, and his only medication is as-needed ibuprofen for his joint pain.On PE, vital signs are normal. The right third DIP joint is swollen, with localized tenderness to palpation and pain with active and passive range of motion. The appearance of the nails is shown in picture. The left second toe shows fusiform swelling and mild diffuse tenderness with decreased active and passive range of motion. There is onycholysis of several toenails, including the left second toenails. The remainder of the examination is normal.Which of the following is the most likely diagnosis? A. Lyme arthritis B. Osteoarthritis C. Psoriatic arthritis D. Rheumatoid arthritis

C. Psoriatic arthritis

7. A 19-year-old boy is brought to the physician by his mother because of a 2-year history of increasingly withdrawn behavior. She says he spends most of his time in his room using his computer and playing video games. He does not have any friends and has never had a girlfriend. He says family dinner is boring and refuses to attend; he prefers being alone. When his mother starts crying during the visit, he appears indifferent. Physical and neurologic examinations show no other abnormalities. Mental status examination shows that his thought process is organized and logical. His affect is flattened. Which of the following is the most likely diagnosis? A. Schizophreniform disorder B. Schizotypal personality disorder C. Schizoid personality disorder D. Antisocial personality disorder E. Avoidant personality disorder

C. Schizoid personality disorder

A 34-year-old man comes to the physician because of increasing lower back and neck pain for the past 7 months. The pain is worse in the morning and improves when he plays basketball. He has noticed shortness of breath while playing for the past 2 months. He is sexually active with two female partners and uses condoms inconsistently. He appears lethargic. His vital signs are within normal limits. Physical examination of the back show tenderness over the sacroiliac joints. Range of motion is limited by pain. The lungs are clear to auscultation. Chest expansion is decreased on full inspiration. His leukocyte count is 14,000/mm3 and erythrocyte sedimentation rate is 84 mm/h. An x-ray of the spine shows erosion and sclerosis of the sacroiliac joints and loss of spinal lordosis. Further evaluation of this patient is most likely to show which of the following? A. Hyperextension of the proximal interphalangeal joints with flexion of the distal interphalangeal joints B. Nail pitting and separation of the nail from the nailbed C. Tenderness at the Achilles tendon insertion site D. Ulnar deviation of the fingers bilaterally E. Scaly, plaque-like lesions on the dorsal side of the elbows

C. Tenderness at the Achilles tendon insertion site

A 62-year-old woman comes to the physician because of a 3-day history of itchy blisters on her trunk. She has a 6-year history of hypertension. She recently switched her antihypertensive to an ACE inhibitor. Physical examination shows tense bullae on her trunk and upper limbs. Histologic examination of a skin biopsy of the lesion shows splitting at dermoepidermal junction with eosinophilic infiltration. Her condition is most likely an example of which of the following types of hypersensitivity reactions? A. Type 3 hypersensitivity B. Type 1 hypersensitivity C. Type 2 hypersensitivity D. Type 4 hypersensitivity-immediate type E. Type 4 hypersensitivity-delayed type

C. Type 2 hypersensitivity

31-year-old man comes to the physician because of a 9-month history of cauli-flower like painless skin lesions in his hands and perineal region. Physical examination shows verrucous (fungating) papules and plaques in his fingers, penis and peri-anal region as shown in the figure. A. Actinic keratosis B. Seborrheic keratosis C. Verruca vulgaris (warts) D. Squamous cell carcinoma E. Sebaceous gland hyperplasia F. Lichen planus

C. Verruca vulgaris (warts)

34-year-old woman comes to the physician with 3-month history of fatigue, arthritis and pruritic skin rashes. There is a 2-month history of weakness in upper limbs with difficulty in combing and reaching for things kept at a height. Physical examination shows violaceous erythema on the periorbital skin as well as posterior neck, upper back, and shoulders. Laboratory studies show increased levels of creatine kinase and high titers of anti-Jo 1 antibodies. Which of the following is the most likely additional finding in this patient? A. Erythematous facial rash with sparing of nasolabial folds B. Positive Schirmer's test C. Violaceous papules on dorsal surface of MCP joints D. Positive Edrophonium test E. Diffusely puffy fingers

C. Violaceous papules on dorsal surface of MCP joints

Which patient would have no difference between their calculated osmolarity and measured Osmolarity? A. A patient in septic shock B. A patient in DKA C. A patient who ingested Antifreeze D. A patient 4 hours after Tylenol overdose E. A patient with Severe Dehydration

D. A patient 4 hours after Tylenol overdose

A 21-year-old woman comes to the physician because of a 7-month history of a skin lesion, menstrual irregularities and headache. Her temperature is 37°C (98.6°F) and blood pressure is 164/100 mm Hg. Physical examination shows increased body hair and facial hair, and comedones on face, neck and shoulders. Some of these lesions are filled with pus. Laboratory studies confirm the diagnosis of congenital adrenal hyperplasia. Elevated serum level of which of the following hormones most likely initiates the pathogenesis of this patient's skin lesion? A. Cortisol B. Insulin C. Aldosterone D. Androgens E. Progesterone

D. Androgens

Which one of the following is a basic activity of daily living ? A. Transportation B. Shopping C. Cooking D. Bathing E. House cleaning and laundry

D. Bathing

2. An 81-year-old man with metastatic prostate cancer comes to the physician requesting assistance in ending his life. He states: "I just can't take it anymore; the body pain is unbearable. Please help me die." His current medications include codeine and Motrin, his cancer has progressed despite chemotherapy, and he is very frail. He lives alone. Which of the following is the most appropriate initial action by the physician? A. Submit a referral to psychiatry B. Submit a referral to hospice care C. Consult with the local ethics committee D. Change his medication to hydrocodone E. Initiate authorization of physician-assisted suicide

D. Change his medication to hydrocodone

A 35-year-old man comes to the physician because of a 5-month history of an enlarging ulcer in his right leg. It occurred at the site where he had removed a wood splinter 6 months ago. Physical examination shows an enlarged cauliflower-like papule with a central ulceration. His right leg is edematous. Microscopic examination of a KOH preparation of the scrapings from the lesion shows pigmented hyphae and Medlar bodies. Which of the following is the most likely diagnosis? A. Cutaneous warts B. Necrotizing fasciitis C. Verrucous carcinoma D. Chromoblastomycosis E. Dermatophyte fungal infection

D. Chromoblastomycosis

Clicker Question: A 39-year-old man comes to the physician for an annual visit. He is currently asymptomatic. His father died at age 55 from heart failure and had uncontrollable dance-like movements for 10 years. His paternal grandfather, aunt, and cousin have similar uncontrollable movements. The patient's DNA report shows >36 CAG trinucleotide repeats in the HTT gene. Which of the following most likely caused the dance-like movements in the patient's father? A. Increased glutaminergic input to the thalamus B. Decreased glutaminergic input to the striatum C. Increased GABA-ergic input to the substantia nigra D. Decreased GABA-ergic input to the thalamus

D. Decreased GABA-ergic input to the thalamus

A 16-year-old girl comes to the physician to request an abortion. She explains she had consensual, unprotected sex with her boyfriend approximately 9 weeks ago. She says she usually has a good relationship with her parents, but they did not approve of her new boyfriend. Now she feels embarrassed that she did not use protection. A serum pregnancy test is positive. Ultrasound shows an intrauterine pregnancy consistent in size with an 8-week gestation. The physician wife experienced multiple early pregnancy losses after many years of trying unsuccessfully to have a child. He no longer performing abortions on his patients. Which of the following is the most appropriate initial action by the physician? A. Explain that he cannot perform abortions because of personal reasons and request that the patient find a different doctor. B. Inform the patient that she presents past the gestational age limit for legal abortion. C. Share his personal situation with the patient and encourage her to think about the decision. D. Encourage the patient to talk to her parents about her pregnancy and her wish for an abortion. E. Explain to the patient that he is only obligated to perform an abortion with parental consent if it is a case of rape or incest.

D. Encourage the patient to talk to her parents about her pregnancy and her wish for an abortion.

A 15yo woman with PMHx of type one diabetes, poorly controlled and JIA (Juvenile idiopathic Arthritis) on medications including Methotrexate, Folic acid, Tocilizumab, Nsaids, recently finished a long course of steroids, Novolog and Levemir presented to the Emergency room with hemoptysis, head ache, tachycardia, tachypnea, hypotension, desaturations and fever 104'F (40'C) . As you start respiratory support blood work is sent and shows: You recognize this might be septic shock, DKA, and/or MAS with Pulmonary hemorrhageIn the Initial resuscitation she received 100 ml/kg in fluid boluses in the first 24 hours, epinephrine, norepinephrine, Milrinone, sedation and insulin continuous drips, was intubated on the ventilator with high PEEP due to active pulmonary hemorrhage, 4 units of PRBCs, 12 units FFP, - now with anasarca. You started a Lasix drip for diuresis along with intermittent hydrochlorothiazide. On hospital day 4, She develops severe diaphragmatic weakness based on the NAVA catheter feedback synchronizing the ventilator with her breaths and is hypoventilating. What is the most likely electrolyte imbalance? A. Hypermagnesemia B. Hyponatremia C. Hypercalcemia D. Hypokalemia E. Hypophosphatemia.

D. Hypokalemia

A 32-year-old woman is admitted to the intensive care unit because of septic shock. Treatment with fluids is started. She received 100 ml/kg fluid boluses in the first 24 hours. 1 day later, she develops swelling of the lower extremities and abdominal distension. A Lasix (furosemide) drip is started for diuresis. Subsequently, she develops severe weakness and begins to hypoventilate. An ECG shows T wave inversion, ST segment depression, and prominent U wave.Which of the following laboratory findings is most likely in this patient? A. Hypomagnesemia B. Hypocalcemia C. Hyponatremia D. Hypokalemia E. Hypoglycemia

D. Hypokalemia

3. A 25-year-old man is brought to the physician by his female roommate because of a change in his behavior. She states that he has been sleeping only 2-4 hours a night for the past five days while preparing for an upcoming exam. He studies and exercises constantly and is more talkative than usual. His physical exam and toxicology screen are normal. Which of the following is the most likely diagnosis? A. Bipolar I disorder B. Bipolar II disorder C. Delusional Disorder D. Hypomanic episode E. Manic episode

D. Hypomanic episode

6. A 28-year-old woman comes to the physician because of a 4-week history of feeling hopeless, lack of sleep, loss of appetite, and difficulty concentrating. She gave birth to her first child six months ago and attributes these symptoms to have to stay up most nights to care for her baby. She sometimes feels angry and bursts into tears for no apparent reason. She used to enjoy reading but finds no joy in this presentation. The pregnancy and delivery were uncomplicated. A sleep electroencephalogram (EEG) of this patient is most likely to show an increase in which of the following? A. Increase REM latency B. Increase in non-REM N3 sleep C. Increase delta wave sleep D. Increased REM density E. Normal EEG reading

D. Increased REM density

11. A 23-year-old man is brought to the emergency department from a college party because of a 1-hour history of a crawling sensation under his skin and chest pain. He appears anxious and is markedly pale. His temperature is 38°C (100.4°F), pulse is 104/min, respirations are 18/min, and blood pressure is 155/90 mm Hg. Physical examination shows diaphoretic skin, moist mucous membranes, and dilated pupils. Which of the following is the most likely primary mechanism responsible for this patient's symptoms? A. Stimulation of μ-opioid receptors B. Direct stimulation of GABA receptors C. Stimulation of cannabinoid receptors D. Inhibition of norepinephrine and dopamine reuptake E. Inhibition of NMDA receptors

D. Inhibition of norepinephrine and dopamine reuptake

A 36-year-old woman comes to the physician because of a 6-month history of a growing skin lesion in the front of her left knee. She says it started as a 2-mm lesion when she started scratching an old scar because of itchiness, 6 months ago. She had a cut in her left knee which healed with a scar, 2 years ago. Physical examination shows a 16-mm silvery white scaly plaque on an erythematous base. Which of the following triggering factors is most likely responsible for the pathogenesis of this patient's condition? A. Age B. Mental stress C. Alcoholism D. Koebner phenomenon E. Family history

D. Koebner phenomenon

A 5-year-old boy is brought to the physician by his mother because of a 2-hour history of a cut in his forearm due to fall. Physical examination shows a 1-cm shallow cut on the back of his right upper forearm near the elbow. Which of the following cells in the epidermis is most likely active in antigen uptake, processing and presentation to naive T cells? A. Keratinocyte B. Microglial cell C. Kupfer cell D. Langerhans cell E. Melanocyte

D. Langerhans cell

A 10-year-old boy is brought to the physician by his mother because of a 10-day history of fever, joint pain and skin rash over legs. He had sore throat which resolved with a course of antibiotics, 1 month ago. His temperature is 37.2°C (99°F), pulse is 92/min, respirations are 22/min and blood pressure is 130/80 mm of Hg. Physical examination shows palpable purpura on legs and trunk. Laboratory studies show RBC casts in urine. Which of the following renal biopsy findings are most likely in this patient? A. Subendothelial deposits; granular IgG and C3 B. Mesangial deposits; granular IgG and C3 C. Subendothelial deposits; granular IgA D. Mesangial deposits; granular IgA E. Subepithelial deposits; granular IgG and C3

D. Mesangial deposits; granular IgA

A 37-year-old man comes to the physician because of a 1-month history of painless skin lesions on the dorsum of his hands and left thumb. He works as a chef in the local restaurant. He does not have any medical conditions. Physical examination shows multiple fungating papules and plaques in the dorsum of his hands that have sponge appearance. Histologic examination of a biopsy of the lesion shows acanthotic epidermis with papillomatosis, hyperkeratosis and parakeratosis. Which of the following predisposing factors most likely made it easy for the inoculation of the etiologic agent? A. Low cell mediated immunity B. Congenital epidermal defect C. Contact with meat D. Micro abrasions E. Multiple sexual partners

D. Micro abrasions

A 56-year-old man comes to the physician because of a 1-year history of recurrent episodes of burning and stinging sensation and flushing of face. He says the episodes are triggered by hot showers or spicy food. Physical examination shows erythematous papules, pustules, and telangiectasias on forehead, cheeks, nose and chin. He also has rhinophyma of nose and otophyma of ears. Laboratory studies show normal serum 5-HT level. Which of the following additional complications is most likely in this patient? A. Hypertension B. Glomerulonephritis C. Gangrene of fingers D. Ocular complications E. Esophageal cancer

D. Ocular complications

6. A 35-year-old man comes to the physician because of a 1-year history of a non-healing ulcer in his right leg which is resistant to treatment. The lesions started as a small, pink, scaly papule 1 year ago, and is now a wart-like ulcer. He lives in the tropics. He says he took multiple courses of antibiotics which did not work. His vital signs are all within normal limits. Physical examination shows a sharply demarked chronic ulcer with indurated base and red cauliflower-shaped nodular projections. Numerous black dots are seen on the surface of the ulcer. Histopathologic examination of a biopsy of this lesion shows a granulomatous reaction with giant cells, and a mixed dermal infiltrate. Which of the following factors of the etiologic agent is most likely responsible for the indolent nature of his condition? A. Lipid and glycogen structures surround the pathogen B. Pathogen develops a cell wall resistant to phagocyte invasion C. Pathogen secretes a proteolytic enzyme that kills phagocytes D. Pathogen develops a sclerotic form after inoculation E. Hydrophobicity of the pathogens reduces cell-mediated host immunity

D. Pathogen develops a sclerotic form after inoculation

5. A 6-year-old boy is brought to the emergency department because of severe crampy abdominal pain and bilious vomiting for 4 hours. The pain started suddenly, and he is not passing any gas or stool for 24. His temperature is 37.8°C (100°F) and pulse is 122/min. Physical examination shows an ill-looking boy with distended abdomen. There is tenderness to palpation in the lower abdomen; guarding and rebound tenderness are present. Bowel sounds are decreased. An x-ray of the abdomen shows dilated loops of bowel and scattered air-fluid levels. He has been accompanied by his 16-year-old sister. The surgeon recommends an emergency laparotomy. The parents cannot be reached at this moment. Which of the following is the most appropriate next best step in management? A. Get consent from the patient's brother B. Obtain a court order for surgery C. Schedule hospital ethics consult D. Perform emergency laparotomy E. Delay surgery until parental consent

D. Perform emergency laparotomy

You are called to the ED to examine a previously healthy 6-year- old boy who has hypotonic dehydration. He has no history of vomiting or diarrhea, and his mother notes that he always has been drawn to salty foods and uses the saltshaker liberally. Examination of an old chart reveals that this is the child's second episode of hypotonic dehydration in the past 2 years, the previous one being last summer. His urinary sodium concentration is less than 5 mEq/L (5 mmol/L). After treating his acute issue, what would be the next step? A. Administer daily furosemide B. obtain a brain MRI C. start on a carbohydrate free diet D. Refer for sweat test E. Recommend aggressive oral hydration with H20 on hot days.

D. Refer for sweat test

An 11-month-old boy is brought to the physician because of a 1-day history of a skin rash following a 4-day of high fever. Mother states that despite being febrile her son is otherwise well. He is afebrile now. Physical examination shows pink macules and papules on his trunk and proximal extremities as shown in the figure. Which of the following is the most likely diagnosis? A. Measles B. Rubella C. Scarlet fever D. Roseola infantum E. Toxin-mediated erythema (staph, strep) F. Drug rash

D. Roseola infantum

10. A 26-year-old woman was brought to the emergency room because she refused to sleep for more than ten days. She states that she has all the energy in the world. She says she is Lady Gaga's daughter and is a very important person who knows many famous actors. At times, she is observed directing her attention to random parts of the room, presumably due to active auditory hallucinations. She also believes she only needs to think about something to make it happen. Three months ago, she was seen by her physician for a six weeks history of "seeing spirits in her home." She also heard voices conversing during that time, yelling or giving her directions, and even telling her to kill herself. On examination, she was sitting in a somewhat provocative position on the couch. Her speech was high-pitched. She appeared relaxed, although, at random times, she would get tense. Her thought process is tangential, with intermittent involuntary silences mid-speech. Her attention was severely diminished. Which of the following is the most likely diagnosis? A. Brief psychotic disorder B. Schizophreniform disorder C. Bipolar I disorder with psychotic features D. Schizoaffective disorder E. Schizophrenia F. Bipolar II disorder with psychotic features

D. Schizoaffective disorder

A 7-year-old boy is brought to the physician by his mother for the evaluation of abdominal pain and trouble sleeping for the past 6 months. His mother says he complains of crampy abdominal pain every morning on school days. He started attending a new school 7 months ago and she is concerned because he has missed school frequently due to the abdominal pain. He also has trouble falling asleep at night and asks to sleep in the same room with his parents every night. He does not have fever, vomiting, diarrhea, or weight loss. He sits very close to his mother and starts to cry when she steps out of the room to take a phone call. Abdominal examination shows no abnormalities. Which of the following is the most likely diagnosis? A. Normal behavior B. Social anxiety disorder C. Conduct disorder D. Separation anxiety disorder E. Acute stress disorder

D. Separation anxiety disorder

Mental status examination shows a neatly dressed, cooperative patient with a depressed mood and affect, and she demonstrated psychomotor retardation.Which of the following is the most likely explanation for the patient's symptoms? A. Illness anxiety disorder B. Malingering C. Major depressive disorder D. Somatic symptom disorder E. Conversion disorder

D. Somatic symptom disorder

A 26-year-old man comes to the physician because of a 3-month history of a non-itchy skin lesion. Physical examination shows hypopigmented macules and patches with fine scales in his trunk as shown in the figure. Which of the following is the most likely diagnosis? A. Tinea capitis B. Tinea manuum C. Tinea pedis D. Tinea versicolor E. Tinea unguium

D. Tinea versicolor

3.A 25-year-old man is brought to the emergency department by his mother because of erratic behavior over the past six months. He spends most of his time alone in his room because he believes he is being followed by aliens. He appears suspicious of his surroundings and asks the doctor questions about "the security of the hospital." The patient exhibits a flat affect. During the physical examination, he tells the doctor that he has a gun at home and plans to kill his neighbor, whom he thinks is working with the aliens. Which of the following is the most appropriate action by the doctor? A. Administer an antipsychotic and request a psychiatric consultation B. Discuss the diagnosis and therapy plan with the patient's mother C. Inform security and law enforcement D. Warn the person at risk and inform law enforcement E. Request a court order to override the patient's right to confidentiality

D. Warn the person at risk and inform law enforcement

What is the BEST way to Assess Acute Water Loss? A. History B. Vital Signs C. Urinalysis D. Weight Change E. Hematocrit F. Physical exam

D. Weight Change

A 43-year-old woman comes to the physician because of a 3-week history of progressive weakness. She has had increased difficulty combing her hair and climbing stairs. She has hypertension. She has smoked one pack of cigarettes daily for 25 years. She does not drink alcohol. Her mother had coronary artery disease and systemic lupus erythematosus. Her current medications include chlorthalidone and vitamin supplements. Her temperature is 37.8°C (100.0°F), pulse is 71/min, and blood pressure is 132/84 mm Hg. Cardiopulmonary examination shows no abnormalities. Skin examination shows diffuse erythema of the upper back, posterior neck, and shoulders. Which of the following antibodies are most likely to be present in this patient? A. Anti-centromere antibodies B. Anti-Ro antibodies C. Anti-histone antibodies D. Voltage-gated calcium channel antibodies E. Anti-Jo-1 antibodies F. Anti-myeloperoxidase antibodies G. Anti-Scl-70 antibodies

E. Anti-Jo-1 antibodies

10. A 70-year-old man is brought to the emergency department after suffering a car accident. The patient suffered a hyperextension of the neck. Physical examination shows muscle strength 2/5 bilaterally in the upper extremities. Muscle strength in the lower extremities was intact. There is loss of pain and temperature in the upper extremities. Cervical spine MRI is shown in the image. Which of the following additional findings is most likely in this patient? A. Loss of pain and temperature sensation in the lower extremities B. Loss of vibration and proprioception in the lower extremities C. Loss of vibration and proprioception in the upper extremities D. Hyperreflexia and hypertonia in both upper extremities E. Burning sensation in the upper extremities

E. Burning sensation in the upper extremities

9. A 66-year-old man is brought to the hospital by paramedics following a car crash. He is conscious but is not able to move his lower limbs. Physical examination shows complete loss of sensation and movement below T10. He has normal sensation and power in the upper limbs. He cannot sit in the bed, lies in a supine position but can move his head. If proper nursing care is not provided, this patient is most likely to develop pressure ulcers in which of the following body parts? A. Buttocks and toes B. Back of head, buttocks and heels C. Chest and heels D. Back of head and chest E. Buttocks and heels

E. Buttocks and heels

13. A 28-year-old woman is brought to the emergency department by her husband because of a 2-day history of multiple cut marks on the wrists. She attempted suicide in the past and was diagnosed with major depression 5 years ago for which she is taking antidepressants. Her mother was diagnosed with major depressive disorder at age 30 and passed away in a car crash 2 years ago. Physical examination shows sudden, dance-like movements of her arms and slow, writhing movements of her fingers. An MRI of the brain is shown on the right. Which of the following cerebral nuclei is most likely affected in this patient and her family? A. Substantia nigra B. Internal globus pallidus C. External globus pallidus D. Thalamus E. Caudate nucleus

E. Caudate nucleus (Huntington's disease)

A 69-year-old woman is brought to the emergency because of a 7-hour history of severe headaches and confusion. She has a history of 2 similar episodes in the past. Her vitals are normal. Physical examination shows a Glasgow coma scale of 9 (normal is 15). Non contrast CT scan of the head reveals multiple hemorrhages restricted to the cortical region with no other diagnostic lesion. MRI SWI sequence shows disseminated cortical microbleeds sparing the basal ganglion and the brainstem. Which of the following is the most likely cause of this patient's pathology? A. Meningeal artery rupture B. Intracranial tumor C. Saccular berry aneurysm D. Venous rupture E. Cerebral amyloid angiopathy

E. Cerebral amyloid angiopathy

A 48-year-old man is admitted to the intensive care unit because of an intracranial hemorrhage. His non contrast CT scan at admission is shown.External ventricular drainage and clipping of the aneurysm is done.On day 14 of his admission, his urine output is very high.Laboratory studies show elevated urine osmolality and elevated urine sodium.Serum sodium is 122mEq/L(normal 136-146). Which of the following is the most likely cause of these urinary findings ? A. SIADH B. Psychogenic polydipsia C. Central diabetes insipidus D. Nephrogenic diabetes insipidus E. Cerebral salt wasting

E. Cerebral salt wasting

On further discussion with her physicians, she stated that she had recently moved to the state to start graduate school; she was excited to start her studies and get her career on track. She was concerned about the impact her seizures might have on her long-term health and was worried that a protracted hospitalization might cause her to miss classes. She was quite concerned about the costs of her hospitalization. When the findings of the vEEG study were discussed with her, she quickly became quite irritable, asking, "So, everyone thinks I'm just making this up? She then pulled her EEG leads from her scalp, dressed, and left the hospital against medical advice. Which of the following is the most likely explanation for this patient's symptoms? A. Illness anxiety disorder B. Malingering C. Major depressive disorder D. Somatic symptom disorder E. Conversion disorder

E. Conversion disorder

A 45-year-old man was brought to the emergency room after a motor vehicle accident that resulted in an open fracture of the lower right limb. He has a history of alcohol abuse and liver disease. Four days after his hospitalization, he complained of bugs crawling on his skin and became confused and disoriented. His blood pressure is 165/102 mm Hg, pulse is 130/min, respirations are 19/min, and temperature is 38.6°C (101.1°F). On physical examination, the patient is diaphoretic and tremulous. Which of the following is the most likely diagnosis? A. Alcohol intoxication B. Withdrawal shakes C. Withdrawal Seizures D. Alcohol Hallucinosis E. Delirium tremens

E. Delirium tremens

1. A 19-year-old man is brought to the emergency department by his father because, over the past two months, he has been getting progressively more paranoid. He states he has to wear a special hat before leaving home for work because the CIA is spying on him and trying to control his mind. He smoked marijuana frequently in high school but quit two years ago. Physical and neurologic examinations show no abnormalities. On mental status examination, his mood is anxious with an appropriate affect. His speech is organized and coherent. Which of the following is the most likely diagnosis? A. Brief psychotic disorder B. Schizoaffective disorder C. Schizophrenia D. Schizophreniform disorder E. Delusional disorder

E. Delusional disorder

6. A 50-year-old woman is brought to the physician because of a 3-month history of progressive visual impairment and headaches. The patient has a 5-year history of hypertension with adequate control. The headaches occur typically in the mornings. Physical examination shows changes in visual acuity. Fundoscopic exam shows mild papilledema. Brain MRI is shown on the right. Surgical removal of the lesion was performed, and pathology report showed a whorled pattern, psammoma bodies, and calcifications. Which of the following is most likely a characteristic of the lesion in this patient? A. Children are more affected than adults B. Origin from oligodendrocytes C. Origin from Schwann cells D. Male predominance E. Dura attachment F. Ventricular location

E. Dura attachment

A 66-year-old woman with terminal breast cancer comes to the emergency department because of difficulty breathing for the past 2 days. Two weeks ago, chest CT scan showed lung metastases. At the time, she declined additional treatment and provided a written advance directive that designated medical power of attorney to eldest son in the event that she lacked the capacity to make medical decisions. The patient appears lethargic and thin. On mental status examination, she is alert and cooperative. She expresses that she does not want to receive any lifesaving procedures. She accurately describes her medical history and understands the consequences of her decision. The patient's eldest son, who is in the emergency department, says that he wants his mother to receive all necessary measures to stay alive. Which of the following is the most appropriate action by the physician? A. Follow the son's wishes B. Ask the patient to reconsider her decision C. Discharge the patient against medical advice D. Obtain a psychiatric consultation E. Follow the patients orally expressed wishes

E. Follow the patients orally expressed wishes

3. A 5-year-old boy is brought to the physician because of a 3-month history of multiple painless skin lesions. There is no history of fever. Physical examination shows multiple rounded, dome-shaped papules with central umbilication on his abdomen, inner thighs and face. Which of the following histologic findings is most likely in a skin biopsy of the lesion? A. Epidermal acanthosis and hyperkeratosis B. Dysplastic keratinocytes C. Multinucleated giant cells with epidermal acantholysis D. Epidermal spongiosis E. Henderson-Patterson bodies

E. Henderson-Patterson bodies

A 45-year-old woman presents to the clinic with the complaint of nausea, vomiting, muscular weakness, dysuria, and generalized pruritis. She is on chemotherapy treatment for Burkitt's lymphoma. Vital signs are a pulse of 78/min, blood pressure of 125/75 mmHg, respiratory rate of 17/min, and temperature of 98.6 F. Physical examination shows bilateral rhonchi on the lung auscultation and grade 1 pedal edema. Which electrolyte abnormality will you Find and need to emergently address? A. Hypokalemia B. Hypercalcemia C. Hypomagnesemia D. Hyponatremia E. Hyperphosphatemia

E. Hyperphosphatemia

15. A 25-year-old woman comes to the physician because of a 1-week history of intermittent, stabbing, facial pain. The pain occurs in waves lasting one second each. The pain is bilateral but rarely occurs on both sides simultaneously. Touching her face or brushing her teeth causes facial pain. Four months ago, she had right arm weakness and numbness on the fingers of the right hand that lasted for one week with complete recovery. Physical examination shows decreased sensation in the V2 and V3 distribution of her face bilaterally, power 3/5 in the left upper extremity and 5/5 in the right upper extremity. There is spasticity and clonus in the lower extremities. Brain MRI is shown. Which of the following additional findings is most likely in this patient? A. Enhancing lesion in the vestibular nerve in brain MRI B. Albuminocytologic dissociation in the CSF C. Schwann cell damage D. Reed-Sternberg cells E. Improvement and recurrence of the symptoms

E. Improvement and recurrence of the symptoms

2. A 22-year-old man comes to the physician because of generalized fatigue. Over the past two months, he has become increasingly sad; during this time, his grades have declined in his college courses because he has had difficulty focusing on assignments. His energy level is low, and he has difficulty falling asleep. He also states that he has lost about 10lbs in the last month. He has become very withdrawn and does not go hiking with friends, which he loves to do. He states that he often thinks about giving up on life. Mental status examination shows constricted affect. His speech is slow in rate and monotone in rhythm, and his thought process is organized. Which of the following symptoms is the most indicative of major depressive disorder? A. Weight loss B. Sleep disturbance C. Thoughts about giving up on life D. Loss of interest in hiking E. Low energy F. Difficulty concentrating

E. Low energy

8. A 28-year-old woman comes to the physician because of difficulty staying asleep that started seven weeks after her 4-month-old daughter was born. She is easily distracted from normal daily duties and feels sad all the time. She says that she always feels exhausted, lacks appetite, and no longer enjoys social activities. Her pregnancy was complicated by gestational diabetes. She is a single parent. She does not smoke, drink alcohol, or use illicit drugs. Vital signs are within normal limits. She is alert and cooperative but makes little eye contact. Physical examination shows no abnormalities. Which of the following is the most likely diagnosis? A. Persistent depressive disorder B. Postpartum blues C. Adjustment disorder D. Depression with peripartum-onset E. Major depressive disorder

E. Major depressive disorder

9. A 34-year-old woman is brought to the emergency department by ambulance because of dizziness, confusion, and fatigue. She was admitted for few times in the past for a similar episode. The first two episodes started before a court hearing after she was indicted for being an accomplice in a bank robbery. The patient reports normal food intake with regular meals. Pulse is 120/min. Physical examination reveals a mild tremor in both hands. Her blood glucose level is 55 mg/dL. Laboratory investigations show low C-peptide levels. Which of the following is the most likely diagnosis of this patient? A. Conversion disorder B. Factitious disorder C. Illness anxiety disorder D. Somatic symptom disorder E. Malingering

E. Malingering

4. A 27-year-old man is brought to the emergency department because of a 2-hour history of a fall from a height. He is a trapeze artist and fell during practice but was noted to be "okay" immediately after the incident. He subsequently became drowsy and barely responsive, prompting his teammates to take him to the hospital. A non-contrast head CT scan shows a biconvex lesion in the left parietal region. He undergoes evacuation of the hematoma and is treated with a drug that reduces water reabsorption in the renal tubules. Which of the following drugs is most likely administered to this patient? A. Acetazolamide B. Eplerenone C. Furosemide D. Hydrochlorothiazide E. Mannitol

E. Mannitol

A 36-year-old-man comes to the physician because of a 2-month history of a painless bumpy skin lesion near his right eye. There is no history of fever. Physical examination shows a 4-mm dia, non-tender, pearly yellow-pink umbilicated papule in the upper eyelid of his right eye. A. Basal Cell Carcinoma B. Xanthoma C. Amelanotic melanoma D. Sebaceous hyperplasia E. Molluscum Contagiosum F. Pyogenic granuloma

E. Molluscum Contagiosum

A 41-year-old man comes to the physician because of nervousness and difficulty sleeping over the past month. Three months ago, he started a new project at his job. He says that he has trouble falling asleep at night because he thinks about the project. He insists on checking the work of his teammates and spends several hours each week making elaborate lists and schedules. He refuses to let others take on more tasks because he thinks that only he can complete them satisfactorily. The project deadline has already been postponed once. He admitted that since he was a teenager, he has used the majority of his time to plan out daily task and does not have time to regularly participate in social activities. Which of the following is the most likely diagnosis? A. Obsessive-compulsive disorder B. Generalized anxiety disorder C. Schizoid personality disorder D. Adjustment disorder E. Obsessive-compulsive personality disorder

E. Obsessive-compulsive personality disorder

A 65-year-old-man is brought to the emergency department because of a 2-hour history of confusion and decreased responsiveness. He had undergone a major surgery 12 hours ago. Current medications are morphine, pantoprazole and amlodipine. His respirations are 7/min and blood pressure is 110/86mm Hg; other vital signs are within normal limits. Physical examination shows shallow breathing and no response to any stimuli. Which of the following points on the graph most likely represents the patient's acid-base status? A. Point A B. Point B C. Point C D. Point D E. Point E

E. Point E (respiratory acidosis; compensation is metabolic so inc HCO3)

4. A 29-year-old man is brought to the physician because of a 6-month history of rapidly progressive memory loss. This is associated with episodes of muscle twitching and loss of balance. He works with cows. His temperature is 102°F (38.8°C), pulse is 82/min, and blood pressure is 100/70 mmHg. Physical examination shows hypertonia, hyperreflexia, and reduced muscle bulk in the upper and lower extremities. The extensor toe reflex is positive. He cannot walk in a straight line or perform finger to nose test without errors. ABG analysis is within normal limits. CSF analysis shows normal protein and glucose levels with an elevated 14-3-3 protein. An MRI of the brain shows diffuse gyriform cortical signal changes. Which of the following best explains in the pathophysiology of the disease affecting this patient? A. Lipohyalinosis B. Charcot-Bouchard microaneurysms C. Molecular mimicry D. Inflammation of leptomeninges E. PrPc converts into PrPsc F. PrPsc converts into PrPc

E. PrPc converts into PrPsc

11. A 23-year-old woman, gravida 1, para 0, at 12 weeks' gestation comes to the physician because of a 3-day history of fever and maculopapular rash. She did not receive immunization against measles, mumps or rubella in childhood. Her temperature is 39°C (102°F). Physical examination shows erythematous papules and macules on her face, trunk and extremities, and enlarged and tender post-auricular lymph nodes. Laboratory studies show positive Rubella-specific IgM antibodies. Which of the following is the next best step in the management of this patient? A. Termination of pregnancy B. MRI to screen for congenital malformation C. Chorionic villous sampling for histopathology D. Treat with anti-viral agents E. Prenatal counseling

E. Prenatal counseling

14. A 31-year-old woman comes to the physician because of a 3-week history of throbbing headaches causing disturbed sleep. The pain is worse in the morning and is associated with diplopia and nausea. Her temperature is 98.6°F (37°C), pulse is 65/min, and blood pressure is 130/80 mmHg. Physical examination shows an obese woman. Fundoscopy shows papilledema. An MRI of the brain is shown. Lumbar puncture is done with elevation of the opening pressure. Which of the following is the most likely diagnosis? A. Migraine B. Bacterial meningitis C. Subarachnoid hemorrhage D. Brain tumor E. Pseudotumor cerebri

E. Pseudotumor cerebri

A 29-year-old woman is brought to the physician by her father because of a change in her behavior over the past 6 weeks. The father says that his daughter has become increasingly withdrawn; she has not answered any phone calls or visited her family and friends. The patient says that she has to stay at home because a foreign intelligence service is monitoring her. She thinks that they are using a magnetic field to read her mind. Mental status exam shows disjointed thinking. She is anxious and has a flat affect. Which of the following is the most likely diagnosis? A. Schizophrenia B. Delusional disorder C. Brief psychotic disorder D. Schizoid personality disorder E. Schizophreniform disorder

E. Schizophreniform disorder

A 20-year-old man comes to the physician because of a 6-week history of a 3.6-kg (8-lb) unintentional weight loss and a 2-month history of excessive thirst and excessive urination during day and night. His uncle has had similar symptoms from an early age. Laboratory studies show a fasting blood glucose of 460 mg/dL (normal: 70 to 100 mg/dL). The physician explains to him that his T-cells are unable to differentiate between what is self and non-self. Which of the following T-cells is the physician most likely referring to? A. Double positive T cells in thymic cortex B. Double positive T cells in thymic medulla C. Double negative T cells in thymic cortex D. Single positive T cells in thymic cortex E. Single positive T cells in thymic medulla

E. Single positive T cells in thymic medulla

A 32-year-old man comes to the physician complaining of tingling sensation in his toes and progressive weakness in both legs. On questioning, the patient says that he had bloody diarrhea, nausea, vomiting, and cramps 3 weeks ago that lasted for a few days. He also complains of back pain. He has not traveled recently and has not eaten anything out of the ordinary. Physical examination reveals markedly decreased patellar and Achilles tendon reflexes bilaterally. Which of the following is not considered initially for the management of this patient? A. Plasma exchange B. IV immunoglobulin C. Lumbar puncture D. Pulmonary and cardiac monitoring E. Steroids

E. Steroids

A 35-year-old man first began to use and abuse drugs at the age of 18, his drugs of choice were cocaine and heroin. His drug use escalated rapidly and began early on to interfere with his ability to function in the world. He attempted to stop abusing drugs but instead switched substances to the anti-anxiety drug valium and other opiate painkillers. By the time he was 25 years old, he was hospitalized twelve times for detoxification, but each time he was released, he began immediately to abuse drugs again. By his late 20's, he was homeless and has been homeless ever since. Which of the following will be helpful in making a diagnosis of substance use disorder? A. Persistent unsuccessful efforts to cut down B. Strong desire or urge to use the drug C. Persistent social or interpersonal problems D. Use in physically hazardous situations E. Increased amounts to achieve intoxication F. All of the above

F. All of the above

A 52-year-old man is brought to the Emergency Department after sustaining a witnessed tonic-clonic seizure. The patient states he has had a bitemporal dull, constant headache for the last 3 weeks. CT-scan of the brain shows butterfly growth. Which of the following is the most likely diagnosis? A. Meningioma B. Pilocytic astrocytoma C. Medulloblastoma D. Oligodendroglioma E. Ependymomas F. Glioblastoma

F. Glioblastoma

9. A 16-year-old boy is brought to the physician because of a 3-week history of headaches, nausea and vomiting. Physical examination shows peripheral ataxia, dysmetria, intention tremor and nystagmus. Brain imaging is shown. Which of the following is the most likely present in the biopsy of this patient's lesion? A. Homer-Wright rosettes B. Pseudopalisades C. Perivascular pseudorosettes D. Multiple, ring-enhancing lesions E. Psammoma bodies F. Rosenthal fibers

F. Rosenthal fibers

12. A 37-year-old man comes to the emergency department because of a 1-week history of severe pain around the right eye, VAS 10/10 in intensity, and lasting for about 30 minutes. This is associated with rhinorrhea and lacrimation. He had a similar episode 4 months ago. There is no family history of migraine, hypertension, or diabetes. He does not use recreational drugs. Physical examination shows no abnormalities. Imaging of the brain is unremarkable. Which of the following is the best initial drug to abort the pain? A. Amitriptyline B. Beta-blocker C. Carbamazepine D. Sodium valproate E. Antibiotics F. Sumatriptan

F. Sumatriptan

5. A 60-year-old woman is brought to the emergency department immediately after suffering a seizure. The patient has a 3-year history of small cell lung cancer treated with radiotherapy and chemotherapy. Her husband states that she pointed to her right hand as it began twitching rhythmically, then stiffened every muscle, groan, lost consciousness and fell down. Fifteen seconds later the patient became incontinent, and her arms and legs began jerking rhythmically. On physical examination the patient was unresponsive and unconscious, with deep breathings. Head CT-scan showed multiple ring-enhancing lesions. Which of the following is characteristic of the most likely lesion in this patient? A. Location in 4th ventricle B. Infratentorial and solitary lesion C. Most common primary malignant brain tumor in children D. Good prognosis E. Butterfly appearance F. Calcifications G. Poor prognosis

G. Poor prognosis (metastatic brain cancer)

A 30-year-old female schoolteacher comes to your office for evaluation of her hand pain which she has experiencing for approx. 8 weeks. It is associated with 1-hour morning stiffness and gets easily fatigued. She has a history of recurrent knees pain that is relieved by NSAIDs. Which of the following is/are the most likely differential diagnosis? A. Rheumatoid arthritis B. Ankylosing spondylitis C. Gout D. Systemic Lupus Erythematous E. Osteoarthritis

A, D

A 45-year-old woman comes to the physician because of 3-month history of joint pain and numbness in toes and fingers. Physical examination shows non-blanching purpura on trunk and legs. Laboratory studies show microscopic hematuria with dysmorphic RBCs. Which of the following types of vasculitis can most likely be considered in the differential diagnosis? (multiple answers) A. Granulomatosis with polyangiitis B. Churg Strauss syndrome C. Polyarteritis nodosa D. Takayasu arteritis E. Microscopic polyangiitis F. Henoch Schonlein purpura G. Thrombangiitis obliterans

A,B,E,F

3. A 12-year-old boy is brought to the emergency department because of a 5-day history of fever, headache and eye sensitivity to light. The patient has received all the vaccinations. Physical examination shows the patient is lethargic and with neck stiffness. Kerning and Brudzinski sings are present. Lumbar puncture shows evidence of bacterial meningitis. Which of the following lumbar puncture results is most likely in this patient? Predominant cell/protein/glucose/opening pressure A. A. High PMNs, high low high B. B. High lymphocytes, high low high C. C. High lymphocytes, high low high D. D. High lymphocytes, normal normal normal E. E. Normal cell count, normal normal normal

A. A. High PMNs, high low high

10. A 29-year-old woman comes to the physician because of a 2-day history of fever, headache, malaise, fatigue and a non-itchy solitary skin lesion on the back of her neck. She went for a hike in the woods 10 days ago. Her temperature is 39.5°C (103°F). Physical examination shows a non-tender, solitary, 3-cm diameter annular red plaque on the back of her neck. A culture of the skin lesion shows Borrelia burgdorferi. If untreated, which of the following complications is most likely during the late chronic stage of this condition? A. Arthritis B. AV conduction block C. Bell's palsy D. Regional lymphadenopathy E. Cardiomyopathy

A. Arthritis

12. A 43 -year-old woman is brought to the physician by her husband because of irritable mood that started 5 days ago. Since then, she has been staying up late at night working on a project and is more talkative than usual . She is energetic despite sleeping less than 4 hours per day. Her husband reports that she seems easily distracted. She is usually very responsible, but she spent most of her money on clothes this week. She has never had similar symptoms in the past. Previously she had episodes where she felt too fatigued to go to work and slept almost all day. During those times, she cried excessively, was very indecisive, and expressed feelings of worthlessness. Vital signs are within normal limits. Mental status examination shows she is irritable but cooperative, speech is pressured, and linear thought process. Which of the following is the most likely diagnosis? A. Bipolar II disorder B. Bipolar I disorder C. Major depressive disorder D. Dysthymia E. Cyclothymia

A. Bipolar II disorder

8. A 62-year-old man comes to the physician because of a 3-week history of headache and left leg weakness. The patient has a 12-year history of hypertension with poor control. His vital signs show blood pressure 160/92 mmHg, heart rate is 60/min, and respiratory rate is 12/min. Physical examination shows left lower extremity paresis with spasticity and hyperreflexia. Brain MRI shows a primary brain tumor. The patient is admitted to the hospital for surgical intervention but dies the same night due to a heart attack. The brain lesion is shown in the image. Which of the following is most likely present in this abnormal tissue? A. Central necrosis B. Homer-Wright rosettes C. Perivascular pseudorosettes D. Psammoma bodies E. "Fried-egg" appearance of cells F. Calcifications G. Rosenthal fibers

A. Central necrosis

A 45 yo male presented to the hospital with TBI (traumatic brain injury) 1 week ago. To maintain a high serum osmolarity and worsening ICP. Instead of hanging 3%Nacl to maintain his serum sodium therapeutically at 159 meq/L. Mistakkenly, a bag of 0.33%NaCl was hung. This was noticed at shift change 12 hours later when the patient began to have ICP spikes and decerebrate posturing. What is the complication from lowering Serum sodium too rapidly? A. Cerebral edema B. Central pontine myelinolysis C. None, his sodium was too high D. Brady Cardia

A. Cerebral edema

7. A 58-year-old woman comes to the physician because of a history of a low mood and a 1.3-kg (2.9-lb) weight loss. She says she feels sad with frequent episodes of crying, anger, and more frequent tension headaches, particularly during stress. These symptoms started four months after she lost her only daughter in a car accident 16 months ago. She says she has lost interest in her life, has difficulty envisioning a meaningful life without her daughter, and feels disconnected from others. She still has difficulty accepting the death of her daughter. Which of the following is the most likely diagnosis? A. Complicated grief B. Acute stress disorder C. Major depressive disorder D. Adjustment disorder E. Normal grief

A. Complicated grief

8. A 64-year-old man comes to the emergency department because of a 2-day history of right ear pain, blisters on the auricle and right-sided facial weakness. He says that he had chicken pox during childhood. He has a 14-year history of type 2 diabetes mellitus. Physical examination shows right-sided facial paralysis, vesicles on the right auricle and loss of taste in the anterior 2/3 of his right tongue. This patient's condition is best explained by reactivation of a viral infection in which of the following sites? A. Geniculate ganglion B. Trigeminal ganglion C. Otic ganglion D. Dorsal root ganglion E. Ciliary ganglion

A. Geniculate ganglion

A 34-year-old man comes to the physician because of a 1-month history of fatigue, tiredness, and joint pain and a 2-month history of swelling in legs and ankles. His temperature is 37°C (98.6°F), pulse is 85/min, respirations are 16/min and blood pressure is 140/90 mm of Hg. Physical examination shows conjunctival pallor, scleral icterus and +1 bilateral pitting pedal edema. Laboratory studies show hemoglobin concentration of 10.1 g/dL and reticulocyte count of 2.8%, and a high titer of anti-nuclear antibodies (ANAs). A peripheral blood smear is shown in the figure. Which of the following best explains the peripheral blood findings of this patient? A. Immune complex deposition in vessel walls B. Autoantibodies against RBC antigens C. Bone marrow failure D. Defective DNA synthesis E. Defective globin synthesis

A. Immune complex deposition in vessel walls (schistocytes due to MAHA)

4. A 22-year-old woman comes to the physician because of a 4-day history of itchy eruptions in her ear lobes. She says that she has been wearing new earrings for the past week. There is no history of fever or myalgia. Physical examination shows erythematous macules, papules and vesicles in both ear lobes around the earrings. Patch test is positive for nickel. Laboratory studies show a normal serum IgE level. Which of the following cells most likely mediate this hypersensitivity reaction? A. Macrophages and lymphocytes B. Basophils and eosinophils C. Macrophages and basophils D. Lymphocytes and eosinophils E. Basophils and mast cells

A. Macrophages and lymphocytes

A 29-year-old woman comes to the family physician because of difficulty falling asleep and fatigue over the past five weeks. Since separating from her husband two months ago, she has felt increasingly sad and irritable and finds it difficult to concentrate on daily activities. The week prior to the start of her last menstrual period was particularly difficult. Her appetite has decreased over the past month, and she has been feeling worthless. She admitted to drinking three glasses of wine on weekends and has smoked marijuana twice in the last month. She is not currently on any medication. Mental status examination reveals a blunted effect. Which of the following is the most likely diagnosis? A. Major depressive disorder B. Bipolar II disorder C. Substance-induced depressive disorder D. Persistent depressive disorder E. Premenstrual dysphoric disorder

A. Major depressive disorder

A 6-year-old boy is brought to the physician because of a 3-day history of skin eruption which is preceded by a fever, cough, and runny nose. He did not receive childhood vaccinations. Physical examination shows erythematous macules and papules ("morbilliform") as shown in the figure. The lesion blanches on applying gentle pressure. Which of the following is the most likely diagnosis? A. Measles B. Rubella C. Roseola infantum D. Drug rash E. Toxin-mediated erythema

A. Measles

A 5-year-old girl is brought to the physician by her mother because of a 3-month history of excessive urination and muscle weakness. The mother says she is drinking large amounts of water and is always thirsty. There is a history of kidney disease in her father. Her weight is in 10th percentile. Blood pressure is 100/60 mm Hg. Laboratory studies show hyper-reninemia. Urinalysis shows osmolality of 110 mOsm/kg H2O (normal 50-1200 mOsmol/kg H2O) and a calcium excretion of 600 mg/day (normal100-300 mg/day). Audiometry shows sensorineural hearing loss.Which of the following additional laboratory findings is most likely in this patient? A. Metabolic alkalosis B. Respiratory alkalosis C. Hyperkalemia D. Metabolic acidosis E. Hypercalcemia

A. Metabolic alkalosis -Bartter Syndrome

A 30-year-old man sustains flash burns secondary to a gas line explosion 4 months ago. Two months after the explosion, he begins experiencing recurring thoughts about the accident and disturbing dreams of the explosion. He finds that he is unable to cook on his gas stove, startles easily in response to noises and have difficulty concentrating. His wife is also concern because he no longer likes participating in family activities. Which of the following is the most likely diagnosis? A. Posttraumatic Stress disorder B. Acute stress disorder C. Specific phobia D. Adjustment disorder E. Panic disorder

A. Posttraumatic Stress disorder

A 56-year-old man is brought to the emergency after being found collapsed in the street. His temperature is 36.7⁰C(98.0⁰F), pulse is 40/min, respirations are 10/min, irregular and blood pressure is 158/85 mm Hg. His GCS is E1V3M4 (normal is 15). Physical examination shows right pupillary diameter of 6mm, unreactive to light and left pupillary diameter of 4mm (normal 2-4 mm), reactive to light. He is withdrawing to pain with his right side but not moving his left. NCCT of the head is shown. Damage to which of the following anatomical regions most likely explains his hemiplegia? A. Right cerebral peduncle of midbrain B. Decussation of pyramids in the medulla C. Left cerebral peduncle of midbrain D. Red nucleus in the midbrain E. Dorsal midbrain (tectal plate)

A. Right cerebral peduncle of midbrain

5. A 35-year-old man comes to the physician because of a 6-month history of recurrent headaches described as dull, compressive, non-pulsating, and localized on both sides of his forehead. The headache is triggered by stress or after sleepless nights but is unrelated to physical activity or head movement. There is no history of lacrimation, rhinorrhea, facial sweating, phonophobia, or photophobia. His vital signs are within normal limits. Physical examination shows a man in painful distress with tenderness of the pericranial muscles bilaterally. The patient's symptoms can be best explained by which of the following? A. Sensitization of CNS pain pathways due to prolonged nociceptive stimuli from myofascial structures B. Presence of a specific allergen at his workplace C. Release of calcitonin gene-related peptide at vascular terminals of the trigeminal nerve causing vasodilation D. Elevation in CSF pressure in the absence of obstruction or an intracranial mass lesion E. Inflammation and injury to the optic nerve

A. Sensitization of CNS pain pathways due to prolonged nociceptive stimuli from myofascial structures

A 23-year-old man is brought to the emergency department from a college party because of a 1-hour history of Palpitations. He appears anxious and is markedly pale. His temperature is 38°C (100.4°F), pulse is 104/min, respirations are 18/min, and blood pressure is 155/90 mm Hg. Physical examination shows diaphoretic skin, moist mucous membranes, and dilated pupils. A urine sample was taken to test for illicit substances. His symptoms cleared during observation after reassurance and mild sedation. Which class of substance is responsible for this patient's presentation? A. Stimulants B. Depressants C. Hallucinogens

A. Stimulants

4. A 62 -year-old man is brought to the emergency department because of chest pain on excretion. The patient only speaks and understands Spanish. He is accompanied by his wife who speaks only Spanish and his stepdaughter, who speaks English and Spanish. His pulse is 100/min, respirations are 13/min, and blood pressure is 160/110 mm Hg. ECG and cardiac markers are normal. The physician who cannot speak Spanish, considers administration of antihypertensive and plan for a stress test in the future. Which of the following is the most appropriate action by the physician? A. Wait for a licensed Spanish interpreter to communicate the treatment plan B. Communicate the treatment plan through step-by-step drawings C. Communicate the treatment plan through the stepdaughter D. Perform the treatment without prior communication

A. Wait for a licensed Spanish interpreter to communicate the treatment plan


Kaugnay na mga set ng pag-aaral

Most Common English Words (Set 2)

View Set

Chapter 14 North and South- American History 8th

View Set